Đến nội dung

Hình ảnh

Topic các bộ đề ôn tập và bồi dưỡng học sinh giỏi Toán 9


  • Please log in to reply
Chủ đề này có 90 trả lời

#41
NLT

NLT

    Trung úy

  • Hiệp sỹ
  • 871 Bài viết
Bài 3: Giải hệ phương trình: $\left\{ \begin{array}{l}xy = x + y - z\\yz = 3(y - x + z)\\zx = 2(x - y + z)\end{array} \right.$

Mình là thành viên mới, mình có một cách giải của bài toán này như sau, mong các bạn cho ý kiến :


$\left\{ \begin{array}{l}
xy = x + y - z\,\,\,\,\,\,\,(1) \\
yz = 3(y - x + z)\,\,(2) \\
zx = 2(x - y + z)\,\,(3) \\
\end{array} \right.$

Dễ thấy, nếu xyz=0 thì:
* x=0 => x=y=z=0 hoặc x=0;y=z=6
* y=0 => x=y=z=0 hoặc x=z=4;y=0
* z=0=> x=y=z=0 hoặc x=y=2;z=0
Với xyz khác 0, ta có :

\[
(1) \to z = x + y - xy\,
\]
thế vào (2),(3) ta đc:

\[
\begin{array}{l}
(2) \Leftrightarrow y(x + y - xy) = 3(y - x + x + y - xy) \\
\,\,\,\,\,\,\,\, \Leftrightarrow \,xy + y^2 - xy^2 = 6y - 3xy \\
\,\,\,\,\,\,\,\, \Leftrightarrow \,y(y - xy - 6 + 4x) = 0 \\
\,\,\,\,\,\,\,\, \Leftrightarrow \,y - xy + 4x = 6(vi`\,y \ne 0)\,\,\,\,\,(2') \\
\,\,\,\,\,\,\,\, \\
\end{array}
\]
Bằng việc làm tương tự như trên, ta cũng thế giá trị của z vào (3) và khi đó, ta được phương trình (3'):

\[
x - xy + 3y = 4(3')
\]
Lấy (2') trừ (3') vế theo vế và biến đổi ta được :

\[
y = \frac{{3x - 2}}{2}
\]
Thế giá trị y vừa tìm được vào 1 trong 2 phương trình(2') và (3'), ta được 1 phương trình bậc 2 theo ẩn x hoặc ẩn y, mình làm theo ẩn x và tìm được 2 giá trị của x là \[
\frac{7}{3}
\] và 2 nhưng loại đi giá trị x=2 vì từ đó suy ra y=2 & z=0. Vậy, kết luận, pt có 5 nghiệm (x;y;z)=(0;6;6);(4;0;4);(2;2;0)\[
(0;0;0);(\frac{7}{3};\frac{5}{2}; - 1)
\];
......................
Mình đã đọc cách giải của bạn Phạm Quang Toàn và thấy cách giải đó khá độc đáo, và mình đã cố gắng tìm ra một cách giải khác, đồng ý là không hay bằng cách của bạn Toàn, nhưng mình thường hay cố gắng tìm một những lời giải khác nhau và qua bài toán này, mình xin đưa ra bài toán tổng quát sau

giải hệ phương trình:\[
\left\{ \begin{array}{l}
a_1 xy = b_1 x + \,c_1 y + \,d_1 z) \\
a_2 yz = b_2 y + c_2 x + \,d_2 z) \\
a_3 xz = \,b_3 x + c_3 y + d_3 z) \\
\end{array} \right.
\]

(Với \[ a_i ,b_i ,c_i ,d_i \, \in \,R;\,(i = \overline {1,3} )\] )

............
Có thể nâng lên nhiều nữa nhưng kế tiếp bài toán của bạn Huy là vấn đề trên mình nêu ra.... Mong các bạn cho ý kiến . tks nhiều

Bài viết đã được chỉnh sửa nội dung bởi princeofmathematics: 28-02-2012 - 18:22

GEOMETRY IS WONDERFUL !!!

Some people who are good at calculus think that they will become leading mathematicians. It's funny and stupid.


Nguyễn Lâm Thịnh

#42
tolaphuy10a1lhp

tolaphuy10a1lhp

    Thượng sĩ

  • Thành viên
  • 224 Bài viết

ok, ai post cũng đc, có thêm nhiều vấn đề để trao đổi mà

Mình có đề ST này. thay Huy post cho các bạn làm tạm
Bộ đề số 5 (120')

1) Rút gọn biểu thức:
$\sqrt {14 + 6\sqrt 5 } + \sqrt[3]{{38 - 17\sqrt 5 }}$

2) Giải hệ phương trình:

$\left\{ \begin{array}{l}
\frac{1}{x} - \frac{1}{{x + y + z}} = \frac{1}{2}\\
\frac{1}{y} - \frac{1}{{x + y + z}} = \frac{1}{5}\\
\frac{1}{z} - \frac{1}{{x + y + z}} = \frac{1}{{10}}
\end{array} \right.$

3) a) Cho $\begin{array}{l}
x + y \ge 8;y \ge 3\\
CMR: 27{x^2} + 10{y^3} \ge 945
\end{array}$

b) Cho $a + b = c + d$
CMR : ${c^2} + {d^2} + cd \ge 3ab$

4) Chứng minh rằng phương trình : ${p^4} + {q^4} = {r^4}$
không có nghiệm trong tập hợp các số nguyên tố.

5) Các đường chéo của một tứ giác nội tiếp trong đường tròn ABCD cắt nhau tại E; các điểm K và M là trung điểm các cạnh AB và CD; L và N là hình chiếu của E lên các cạnh BC và AD
. Chứng minh KM vuông góc LN.

6) Trong một kỳ thi HSG có m > 1 thí sinh và giải n > 1 bài toán . Tất cả các thí sinh đã giải một số lượng khác nhau các bài toán. Tất cả các bài toán đều được giải bởi một số lượng thí sinh khác nhau. Chứng minh rằng có một thí sinh giải đúng một bài.

Bài viết đã được chỉnh sửa nội dung bởi tolaphuy10a1lhp: 05-03-2012 - 23:45

Học là ..... hỏi ...............

#43
hangel_elf

hangel_elf

    Hạ sĩ

  • Thành viên
  • 61 Bài viết
Bài 4:
Trước hết,nhận xét thấy lũy thừa bậc 4 của số nguyên khi chia cho 4 chỉ có số dư là 0 hoặc 1.Từ phương trình ban đầu suy ra:
-Cả $q^{4}$ và $p^{4}$ đều chia hết cho 4.Vì p,q là số nguyên tố nên p=q=2.Không thỏa mãn
-Trong 2 số có 1 số chia hết cho 4,một số chia 4 dư 1.Giả sử p chia hết cho 4.Vì p nguyên tố nên p=2.
Ta còn: $16+q^{4} =r^{4}$.
Nhận xét lũy thừa bậc 4 chia 3 chỉ có thể dư 0 hoặc 1.Mà 16 chia 3 dư 1,nên $q^{4}$ phải chia hết cho 3.=>q=3.Nghiệm p=2;q=3 không thỏa mãn.Vậy phương trình vô nghiệm trên tập hợp số nguyên tố

#44
NLT

NLT

    Trung úy

  • Hiệp sỹ
  • 871 Bài viết
Áp dụng ĐL lớn fermat vào bài 4 thôi, không cần chứng minh j` cả, hok chỉ là đối vs số nguyên tố, mà đối vs mọi số nguyên khác 0 nữa kìa

GEOMETRY IS WONDERFUL !!!

Some people who are good at calculus think that they will become leading mathematicians. It's funny and stupid.


Nguyễn Lâm Thịnh

#45
minhtuyb

minhtuyb

    Giả ngu chuyên nghiệp

  • Thành viên
  • 470 Bài viết

3) a) Cho $\begin{array}{l}
x + y \ge 8;y \ge 3\\
CMR: 27{x^2} + 10{y^3} \ge 945
\end{array}$

b) Cho $a + b = c + d$
CMR : ${c^2} + {d^2} + cd \ge 3ab$


Xin chém câu BĐT :D:
a.Áp dụng BĐT AM-GM, có;
$27x^2+675\geq 2\sqrt{27x^2.675}=270x$
$10y^3+270+270\geq 3\sqrt[3]{10y^3.270^2}=270y$
Cộng vế với vế ta có: $27x^2+10y^3+1215\geq 270(x+y)\geq 270.8=2160\Rightarrow 27x^2+10y^3\geq 945<Q.E.D>$
Dấu bằng xảy ra khi $x=5;y=3$

b. Có:
$c^2+d^2+cd=(c+d)^2-cd\geq (c+d)^2-\frac{(c+d)^2}{4}=\frac{3(c+d)^2}{4}=\frac{3(a+b)^2}{4}\geq \frac{3.4ab}{4}=3ab<Q.E.D>$
Dấu bằng xảy ra khi $a=b=c=d$
Phấn đấu vì tương lai con em chúng ta!

#46
nthoangcute

nthoangcute

    Thiếu tá

  • Thành viên
  • 2003 Bài viết
Post này cực kì buồn, mình làm bài 1 này:
_____________________________________
Hình đã gửi

BÙI THẾ VIỆT - Chuyên gia Thủ Thuật CASIO

 

Facebook : facebook.com/viet.alexander.7


Youtube : youtube.com/nthoangcute


Gmail : [email protected]


SÐT : 0965734893


#47
ToanHocLaNiemVui

ToanHocLaNiemVui

    Trung sĩ

  • Thành viên
  • 183 Bài viết


BỘ ĐỀ SỐ 3


Bài 3: Cho pt ẩn x, a là tham số:

${x^4} + 2{x^2} + 2ax + {(a + 1)^2} = 0$

Tìm giá trị min và max trong các nghiệm của pt và a tương ứng.

Mình xem sơ qua hình như chưa ai làm, mình xin phép "bem":
G/s: $x=x_{0}$ là nghiệm của PT, khi đó ta viết PT đã cho thành PT bậc 2 ẩn a:
$a^{2}+2(x_{0}+1)a+x_{0}^{4}+2x_{0}^{2}+1=0$
Để tồn tại a thì PT (*) phải có nghiệm hay:
$\Delta'=(x_{0}+1)^{2}-2(x_{0}^{2}+1)^{2}\geq 0$.
$\Leftrightarrow (x_{0}^{2}+x_{0}+2)(x_{0}-x_{0}^{2})\geq 0$.
$\Leftrightarrow 0\leq x_{0}\leq 1$.
Từ đó: $Min_x=x=x_{0}=0 \Leftrightarrow a=-1$
$Max_x=x=x_{0}=1 \Leftrightarrow a=-2$.

Bài viết đã được chỉnh sửa nội dung bởi ToanHocLaNiemVui: 17-03-2012 - 22:08

Đừng Sợ Hãi Khi Phải


Đối Đầu Với Một Đối Thủ Mạnh Hơn


Mà Hãy Vui Mừng Vì


Bạn Có Cơ Hội Chiến Đấu Hết Mình!

___________________________________________________________________________

Thào thành viên của

VMF


#48
ducthinh26032011

ducthinh26032011

    Thượng sĩ

  • Thành viên
  • 290 Bài viết
Hệ phương trình bài 2 vô nghiệm phải không nhỉ?

Hình đã gửi


#49
henry0905

henry0905

    Trung úy

  • Thành viên
  • 892 Bài viết
Tài liệu ôn tập và bồi dưỡng HSG 9
File gửi kèm  Bo de thi HSG lop 9.pdf   12.98MB   2625 Số lần tải
File gửi kèm  www.MATHVN.com-BD-HSG-Toan9.rar   185.08K   324 Số lần tải

Bài viết đã được chỉnh sửa nội dung bởi henry0905: 03-06-2012 - 00:12


#50
baonguyen97

baonguyen97

    Binh nhì

  • Thành viên
  • 15 Bài viết

Bộ đề số 3

Bài 6: Giải hệ pt:


$\left\{ \begin{array}{l}x + y + xy = 5\\{(x + 1)^3} + {(y + 1)^3} = 35\end{array} \right.$

$xy + x + y = 6$
$\leftrightarrows (x+1)(y+1)=6$
$\leftrightarrows (x+1)^{3}(y+1)^{3}=216$
Đặt $a=(x+1)^{3}$
$b=(y+1)^{3}$
a và b là nghiệm của phương trình: $X^{2}-35X+216=0$
Từ đó giải được hệ.

Bài viết đã được chỉnh sửa nội dung bởi baonguyen97: 07-06-2012 - 11:47


#51
giapvansu

giapvansu

    Binh nhất

  • Thành viên
  • 45 Bài viết

Bài 3: Điều kiện$x\geq 1;x\leq 13$
a) Min
Áp dụng BĐT $\sqrt{a}+\sqrt{b}\geq \sqrt{a+b}$
Dấu "="; xảy ra khi ab=0
Áp dụng ta được$A\geq \sqrt{x-1+13-x}=2\sqrt{3}$
Dấu "="; xảy ra khi x= 1 hoặc x =13
Max
Áp dụng BĐT cauchy-Schwarz
$A^2=(\sqrt{x-1} +\sqrt{13-x})^2 \leq (1+1)(x-1-x+13)=24 \Rightarrow A \leq 2\sqrt{6}$
Dấu "="; xảy ra khi x = 7
Ngoài 2 cách mình nêu trên đây các bạn còn có thể sử dụng phương pháp hàm số để tìm min, max hoặc bình phương 2 vế
b) Áp dụng BĐT Cauchy-schwarz
$B^2\leq (1+1+1)(4(a+b+c)+9)=75 \Rightarrow B\leq \sqrt{75}=5\sqrt{3}$
Dấu bằng xảy ra khi a= b=c=$\dfrac{4}{3}$

Bạn học định thức cấp 2 chưa nhỉ. Nếu học rồi thì bài 6 này giải quyết rất đơn giản.

Bài 6 theo mình hoàn toàn có thể làm theo phương pháp biến đổi quen thuộc trong hệ mà các bạn HS lớp 9 đã được học một cách khá đơn giản
m sẽ nói qua bài 1 một chút, bài còn lại hoàn toàn tương tự
Từ pt thứ nhất ta có thể tính được $x=\frac{1+\left | y \right |}{3}$ rồi thế vào pt thứ hai của hệ
$5\frac{1+\left | y \right |}{3}+3y=11\Leftrightarrow 5\left|y\right|+9y=28$
Đến đây mọi việc trở nên đơn giản rồi phải không nào!
Chúc bạn học tốt!

#52
Tamlun

Tamlun

    Binh nhất

  • Thành viên
  • 29 Bài viết
bạn đưa đề thi học sinh giỏi cấp huyện lên đi

học-học nữa-học mãi-đúp lại

 

 

 

học-đuổi lại xin # Lênin bảo thế !!! :namtay :namtay :namtay


#53
Mai Xuan Son

Mai Xuan Son

    Vagrant

  • Thành viên
  • 274 Bài viết

Áp dụng ĐL lớn fermat vào bài 4 thôi, không cần chứng minh j` cả, hok chỉ là đối vs số nguyên tố, mà đối vs mọi số nguyên khác 0 nữa kìa

Định lí lớn fermat là cả 1 công trình mà bạn,đây là THCS vô thi ghi là áp dụng đl lớn Fermat thì có mà 0 điểm :)
~~~like phát~~~

#54
Zaraki

Zaraki

    PQT

  • Phó Quản lý Toán Cao cấp
  • 4273 Bài viết

BỘ ĐỀ SỐ 4

Bài 1: Giải phương trình nghiệm nguyên: $({x^2} - y)(x - 2y + 1) = (x - 1){(x - y)^2}$

Ta có $$pt \Leftrightarrow 2x^2-x(3y+y^2)+3y^2-y=0.$$
Để phương trình có nghiệm nguyên thì $\Delta_x$ là số chính phương. Do đó $$\begin{aligned} \Delta_x & =(3y+y^2)^2-4 \cdot 2 \cdot (3y^2-y) \\ & = y^4+6y^3-15y^2+8y \\ & = y(y-1)^2(y+8) \end{aligned}$$
là số chính phương khi $y(y+8)$ chính phương. Đặt $y^2+8y=a^2 \Rightarrow (y+4)^2=a^2+16 \Rightarrow (y+4-a)(y+4+a)=16$ với $a \in \mathbb{N}$. Dễ nhận thấy $y+4-a \le y+4+a$ và $y+4-a,y+4+a$ có cùng tính chẵn lẻ nên ta sẽ xét các TH sau:
  • TH1: $\begin{cases}y+4+a=8 \\ y+4-a=2 \end{cases} \Rightarrow y=1 \Rightarrow x=1$.
  • TH2: $\begin{cases}y+4+a=4 \\ y+4-a=4 \end{cases} \Rightarrow y=0 \Rightarrow x=0$.
  • TH3: $\begin{cases}y+4+a=-2 \\ y+4-a=-8 \end{cases} \Rightarrow y=-9 \Rightarrow (x-21)(x-6)=0 \Leftrightarrow \left[ \begin{array} xx=21 \\ x=6 \end{array} \right.$
  • TH4: $\begin{cases} y+4+a=-4 \\ y+4-a=-4 \end{cases} \Rightarrow y=-8 \Rightarrow (x-10)^2=0 \Leftrightarrow x=10$.
Vậy phương trình có nghiệm $$\boxed{(x,y)=(1,1),(0,0),(21,-9),(6,-9),(10,-8)}$$

Discovery is a child’s privilege. I mean the small child, the child who is not afraid to be wrong, to look silly, to not be serious, and to act differently from everyone else. He is also not afraid that the things he is interested in are in bad taste or turn out to be different from his expectations, from what they should be, or rather he is not afraid of what they actually are. He ignores the silent and flawless consensus that is part of the air we breathe – the consensus of all the people who are, or are reputed to be, reasonable.

 

Grothendieck, Récoltes et Semailles (“Crops and Seeds”). 


#55
AnnieSally

AnnieSally

    Thiếu úy

  • Thành viên
  • 647 Bài viết

Bài 6: Giải các hệ phương trình:
1. $\left\{ \begin{array}{l} 3x - \left| y \right| = 1 \\ 5x + 3y = 11 \\ \end{array} \right.$
Chia thành 2 truờng hợp y<0 và y$\geq$0



#56
datcoi961999

datcoi961999

    Thượng sĩ

  • Thành viên
  • 263 Bài viết

BỘ ĐỀ SỐ 1


Bài 1:
2. Giải phương trình: $(x^2 - 2x - 15)\sqrt {16 - x^2 } = 0$(1)
 

ĐK: $-4\leq x\leq 4$
(1)=>$(x-5)(x+3)\sqrt{16-x^2}=0$

=>(1) có 3 nghiệm $-3,-4,4$


                 :dislike    :off: ZION   :off:  :like                                                                                     98efb2f1bfc2432fa006b3d7d9f1f655.0.gif

                                                    


#57
i love math so much

i love math so much

    Binh nhất

  • Thành viên
  • 44 Bài viết

Câu 1:1đRút gọn biểu thức  A=  \frac{\sqrt{1+\sqrt{1-x^{2}}}\left [ \sqrt{(1+x)^{3}} \right\sqrt{(1-x)^{3}} ]}{2+1\sqrt{1-x^{2}}}$

 

Câu 2:2đCHo phương trình: $x^{2}-2(m+1)x+m^{2}+1=0$

a) giai phương trình trên với m=1/($\dpi{120} \frac{1}{2(3-2\sqrt{2})}$

b) tìm m để phương trình có 2 nghiệm x$x_{1};x_{2}$ thỏa mãn $x_{1}^{2}=x_{1}x_{2}+6x_{2}^{2}$

 

Câu 3: 3đ CHo hàm số y=$\frac{-1x^{2}}{2}$ 

a) vẽ đồ thị (p) của hàm số

b) trên (p) lấy hai điểm M và N lần lượt có hoành độ là -2 và -1. Viết phương trình đường thẳng MN

c) Xác định hàm số y= ax+b biết rằng đồ thị d của nó song song với đường thằng MN và chỉ giao với (p) tại một điểm duy nhất

 

Câu 4: 1đ CHo hệ phương trình $\left\{\begin{matrix}xy(x+1)(y+1)=12 & & \\ x+y+x^{2} +y^{2} =8 & & \end{matrix}\right.$

 

Cầu 5: 1đ Giải phương trình $2014x^{4}+x^{4}\sqrt{x^{2}+2014} +x^{2} = 2013.2014$

 

Câu 6:' 2đ Cho đường tròn (O,R) nội tiếp hình thang ABCD (AB// CD) với E ,F,G,F theo thứ tự là tiếp điểm  của (O,R) với các cạnh AB,BC,CD,DA

 

a) Chứng minh: EB.GC=GD.EA từ đó tính tỉ số EB/EA biết AB=4R/3 và BC=3R

 

Câu 7: CHo a,b,c là các thực dương chứng minh rằng

$\dpi{120} \frac{3a^{3}+7b^{3}}{2a+3b}+\frac{3b^{3}+7c^{3}}{2b+3c}+\frac{3c^{3}+7a^{3}}{2c+3a} \geq 3(a^{2}+b^{2}+c^{2}) -(ab+bc+ca)$

 

                       Để thi này cực hay và cũng rất khó mong mọi người thử sức cùng mình. MỌi người giải chi tiết giùm mình với nha!!!!!!!!!



#58
Trang Luong

Trang Luong

    Đại úy

  • Thành viên
  • 1834 Bài viết

đặt a=2014

PT$\Leftrightarrow 2014x^{4}+x^{4}.\sqrt{x^{2}+2014}+x^{2}+2014=2014^{2}\Leftrightarrow ax^{4}+x^{4}.\sqrt{x^{2}+a}+x^{2}+a-a^{2}=0$

$\Leftrightarrow x^{4}(\sqrt{x^{2}+a}+a)+(\sqrt{x^{2}+a}+a)(\sqrt{x^{2}+a}-a)=0\Leftrightarrow (\sqrt{x^{2}+a}+a)(x^{4}+\sqrt{x^{2}+a}-a)\Leftrightarrow x^{4}+\sqrt{x^{2}+a}-a=0$(do a>0)


"Nếu bạn hỏi một người giỏi trượt băng làm sao để thành công, anh ta sẽ nói với bạn: ngã, đứng dậy là thành công"
Issac Newton

#59
Trang Luong

Trang Luong

    Đại úy

  • Thành viên
  • 1834 Bài viết

 

Câu 4: 1đ CHo hệ phương trình $\left\{\begin{matrix}xy(x+1)(y+1)=12 & & \\ x+y+x^{2} +y^{2} =8 & & \end{matrix}\right.$

 

Ta có : $\left\{\begin{matrix} xy(x+1)(y+1)=12 & & \\ x+y+x^{2}+y^{2}=8 & & \end{matrix}\right.\Leftrightarrow \left\{\begin{matrix} x(x+1)y(y+1)=12 & & \\ x(x+1)+y(y+1)=8 & & \end{matrix}\right.$

Đặt $x(x+1)=a,y(y+1)=b$

$\Rightarrow \left\{\begin{matrix} ab=12 & & \\ a+b=8 & & \end{matrix}\right.\Rightarrow\left\{\begin{matrix} a=2 & & \\ b=6 & & \end{matrix}\right.$ hoặc ngược lại


"Nếu bạn hỏi một người giỏi trượt băng làm sao để thành công, anh ta sẽ nói với bạn: ngã, đứng dậy là thành công"
Issac Newton

#60
Quang Minh BG

Quang Minh BG

    Lính mới

  • Thành viên
  • 7 Bài viết

làm thế nào để dowload tài liệu trên VMF về máy hả các bạn






1 người đang xem chủ đề

0 thành viên, 1 khách, 0 thành viên ẩn danh